Quantcast
  • Register
PhysicsOverflow is a next-generation academic platform for physicists and astronomers, including a community peer review system and a postgraduate-level discussion forum analogous to MathOverflow.

Welcome to PhysicsOverflow! PhysicsOverflow is an open platform for community peer review and graduate-level Physics discussion.

Please help promote PhysicsOverflow ads elsewhere if you like it.

News

PO is now at the Physics Department of Bielefeld University!

New printer friendly PO pages!

Migration to Bielefeld University was successful!

Please vote for this year's PhysicsOverflow ads!

Please do help out in categorising submissions. Submit a paper to PhysicsOverflow!

... see more

Tools for paper authors

Submit paper
Claim Paper Authorship

Tools for SE users

Search User
Reclaim SE Account
Request Account Merger
Nativise imported posts
Claim post (deleted users)
Import SE post

Users whose questions have been imported from Physics Stack Exchange, Theoretical Physics Stack Exchange, or any other Stack Exchange site are kindly requested to reclaim their account and not to register as a new user.

Public \(\beta\) tools

Report a bug with a feature
Request a new functionality
404 page design
Send feedback

Attributions

(propose a free ad)

Site Statistics

205 submissions , 163 unreviewed
5,047 questions , 2,200 unanswered
5,345 answers , 22,709 comments
1,470 users with positive rep
816 active unimported users
More ...

  Proof that the superstring action is Weyl invariant

+ 4 like - 0 dislike
2572 views

The superstring action is:

$$ S = k\int \mathrm d\sigma \sqrt{-h} \left [ h^{\alpha \beta} \partial_\alpha X^\mu \partial_\beta X_\mu + 2i {\bar{\psi}} ^\mu \rho^\alpha \partial_\alpha \psi_\mu - i {\bar \chi}_\alpha \rho^\beta \rho^\alpha \psi^\mu \left (\partial_\beta X_\mu -\frac{i}{4}{\bar \chi}_\beta \psi_\mu \right)\right ]$$

This is Weyl invariant, but the symmetry isn't manifest. All the books and lecture notes I've seen just state this outright, without proving it. Is there a reference where I can see the proof done in detail? Even when they give the action in the superconformal gauge ($ h_{\alpha \beta} = \eta_{\alpha \beta}$ and $\chi_\alpha = 0$), they don't bother to prove it.


This post imported from StackExchange Physics at 2014-06-29 09:35 (UCT), posted by SE-user user46242

asked Jun 27, 2014 in Theoretical Physics by user46242 (35 points) [ revision history ]
edited Jul 12, 2014 by Arnold Neumaier
If we take into account quantum corrections, the theory is truly Weyl invariant only for Ricci-flat metrics.

This post imported from StackExchange Physics at 2014-06-29 09:35 (UCT), posted by SE-user JamalS

Related simpler version: physics.stackexchange.com/q/70508

This post imported from StackExchange Physics at 2014-06-29 09:35 (UCT), posted by SE-user joshphysics

@joshphysics I don't think that that is very relevant here. The OP clearly states that he is talking about the superstring action not the bosonic string action, for which, as you state, proving Weyl symmetry is simpler, and much simpler to the degree that mentioning it here is completely useless.

2 Answers

+ 3 like - 0 dislike

Short answer: 1+1=1/2+1/2+1=1/2+1/2+1=1/2+1/2+1/2+1/2=2.

Long answer: the Weyl invariance is (almost) manifest. The proof is essentially a simple dimension counting. As the Weyl symmetry is about rescaling of the metric $h_{\alpha \beta}$, we have to make explicit the metric dependence in the various terms of the action. As the theory contains spinors, we have to use a 2d "tetrad" formalism with a "zweibein" $e^a_\alpha$ satisfying $h_{\alpha \beta} = e^a_\alpha e^b_\beta \eta_{ab}$, $e^\alpha_a e^b_\beta = \eta^b_a$ where $\alpha$, $\beta$... are curved 2d space-time coordinates indices and $a$, $b$... are 2d Minkowski coordinates indices. This formalism is implicit in the action written in the question but it is better to replace the curved metric dependant gamma matrices $\rho^\alpha$ by Minkowski, metric independant, gamma matrices $\rho^a$. So we rewrite the action as:

$S = k \int  d\sigma \sqrt{-h}  ( h^{\alpha \beta}  \partial_\alpha X^\mu \partial_\beta X_\mu + 2i {\bar{\psi}}^\mu \rho^a e^\alpha_a \partial_\alpha \psi_\mu $

$- i {\bar \chi}_a \rho^b \rho^a \psi^\mu e^\beta_b \partial_\beta X_\mu  - \frac{1}{4}{\bar \chi}_a \rho^b \rho^a \psi^\mu {\bar \chi}_b \psi_\mu )$

This action is invariant under the Weyl symmetry

$e^a_\alpha \mapsto \lambda e^a_\alpha$, $X \mapsto X$, $\psi \mapsto \lambda^{-1/2} \psi$, $ \chi \mapsto \lambda^{-1/2} \chi$

where $\lambda$ is an arbitrary function on the 2d spacetime.

Indeed:

As $e^a_\alpha \mapsto \lambda e^a_\alpha$, we have $h_{\alpha \beta} \mapsto \lambda^2 h_{\alpha \beta}$ and so $det(h) \mapsto \lambda^4 det(h)$ because we are in 2d and so $\sqrt{det(h)} \mapsto \lambda^2 \sqrt{det(h)}$. So we have to show that each term between the brackets in the expression giving $S$ is multiplied by $\lambda^{-2}$ under the Weyl symmetry.

First term: term in $h^{\alpha \beta} \partial X \partial X$. As $h_{\alpha \beta} \mapsto \lambda^2 h_{\alpha \beta}$, we have $h^{\alpha \beta} \mapsto \lambda^{-2} h^{\alpha \beta}$ hence the result because $X \mapsto X$.

Third term: term in $\chi \psi e^\beta_b$. We have $\chi \mapsto \lambda^{-1/2} \chi$, $\psi \mapsto \lambda^{-1/2} \psi$, $e^\beta_b \mapsto \lambda^{-1} e^\beta_b$ hence the result because $-1/2-1/2-1=-2$.

Fourth term: term in $\chi \psi \chi \psi$. We have $\chi \mapsto \lambda^{-1/2} \chi$, $\psi \mapsto \lambda^{-1/2} \psi$, hence the result because $-1/2-1/2-1/2-1/2=-2$.

Second term: term in $\overline{\psi} \rho^a e^\alpha_a \partial \psi$. It is the unique slightly subtle point because this term contains a derivative of a field charged under the Weyl symmetry. In fact, it is a general but not completely trivial fact that in any spacetime dimension, a fermion minimally coupled to gravity is (classically) Weyl invariant: the Dirac operator is conformally invariant. Let's see what happens explicitely in our 2d case. Under , $\psi \mapsto \lambda^{-1/2} \psi$, $e^\alpha_a \mapsto \lambda^{-1} e^\alpha_a$, the term  $\overline{\psi} \rho^a e^\alpha_a \partial \psi$ gives

$\lambda^{-2}\overline{\psi} \rho^a e^\alpha_a \partial \psi + \lambda^{-3/2}  \overline{\psi} \rho^a e^\alpha_a \psi \partial (\lambda^{-1/2})$.

We have to show that the undesired extra term vanishes:

$\overline{\psi} \rho^a \psi= 0$,

which is a special property of Majorana fermions in 2d (and which was already implicitely used in the expression of the action $S$ to put an apparently non covariant derivative instead of the full Dirac operator). There are many ways to prove this relation. One possibility: $\overline{\psi} \rho^a \psi = \psi^{T} \rho^0 \rho^a \psi$ because $\psi$ is real; if $a=0$, we obtain something proportional to $\psi^{T} \psi = \psi_{+}^2+\psi_{-}^2=0$ because the components $\psi_{\pm}$ of $\psi$ are anticommuting variables; if $a=1$, $\rho^0 \rho^1$ is the 2d "$\gamma_5$ matrix" and so we obtain something proportional to $\psi_{+}^2-\psi_{-}^2=0$ because the components $\psi_{\pm}$ of $\psi$ are anticommuting variables.

answered Jul 2, 2014 by 40227 (5,140 points) [ revision history ]
edited Jul 2, 2014 by 40227

Wow @40227, thanks a lot for this patiently detailled great answer :-)!

+ 1 like - 0 dislike

As you say, the Weyl symmetry in the superstring action in the RNS formalism is not manifest. This means you can’t really “prove” Weyl symmetry for the superstring action. Instead, you postulate that the superstring action obeys Weyl symmetry (is conformally invariant) and derive the constraint equations necessary for your postulate to be satisfied.

You can an overview of this process in Section 4.3 (pg 118 – 120) of Becker, Becker, and Schwarz, String theory and M-theory. The constraint amounts to just

\[J_-=J+=T_{--}=T_{++}=0\]

answered Jul 1, 2014 by dimension10 (1,985 points) [ revision history ]
edited Jul 1, 2014 by dimension10

Your answer

Please use answers only to (at least partly) answer questions. To comment, discuss, or ask for clarification, leave a comment instead.
To mask links under text, please type your text, highlight it, and click the "link" button. You can then enter your link URL.
Please consult the FAQ for as to how to format your post.
This is the answer box; if you want to write a comment instead, please use the 'add comment' button.
Live preview (may slow down editor)   Preview
Your name to display (optional):
Privacy: Your email address will only be used for sending these notifications.
Anti-spam verification:
If you are a human please identify the position of the character covered by the symbol $\varnothing$ in the following word:
p$\hbar$ysicsOverflo$\varnothing$
Then drag the red bullet below over the corresponding character of our banner. When you drop it there, the bullet changes to green (on slow internet connections after a few seconds).
Please complete the anti-spam verification




user contributions licensed under cc by-sa 3.0 with attribution required

Your rights
...